Hostname: page-component-848d4c4894-ndmmz Total loading time: 0 Render date: 2024-05-12T21:11:11.589Z Has data issue: false hasContentIssue false

Divisor function inequalities, entropy, and the chance of being below average

Published online by Cambridge University Press:  01 March 2017

ZARATHUSTRA BRADY*
Affiliation:
Department of Mathematics, Stanford University, 450 Serra Mall, Bldg. 380, Stanford, CA 94305-2125, U.S.A. e-mail: notzeb@stanford.edu

Abstract

We extend a lower bound of Munshi on sums over divisors of a number n which are less than a fixed power of n from the squarefree case to the general case. In the process we prove a lower bound on the entropy of a geometric distribution with finite support, as well as a lower bound on the probability that a random variable is less than its mean given that it satisfies a natural condition related to its third cumulant.

Type
Research Article
Copyright
Copyright © Cambridge Philosophical Society 2017 

Access options

Get access to the full version of this content by using one of the access options below. (Log in options will check for institutional or personal access. Content may require purchase if you do not have access.)

References

REFERENCES

[1] Alon, N., Huang, H. and Sudakov, B.. Nonnegative k-sums, fractional covers, and probability of small deviations. J. Combin. Theory Ser. B 102 3 (2012), 784796.Google Scholar
[2] FEDJA (http://mathoverflow.net/users/1131/fedja). lower-bound for Pr[XEX]. MathOverflow. URL:http://mathoverflow.net/q/188087 (version: 2014-11-26).Google Scholar
[3] Friedlander, J. B. and Iwaniec, H.. Divisor weighted sums. Zap. Nauchn. Sem. S.-Peterburg. Otdel. Mat. Inst. Steklov. (POMI) 322. Trudy po Teorii Chisel (2005), 212219, 255.Google Scholar
[4] Hartke, S. G. and Stolee, D.. A linear programming approach to the Manickam–Miklós–Singhi conjecture. European J. Combin. 36 (2014), 5370.Google Scholar
[5] Khovanskiĭ, A. G.. Fewnomials. Trans. Math. Monogr. vol. 88 (American Mathematical Society, Providence, RI, 1991). Translated from the Russian by Smilka Zdravkovska.Google Scholar
[6] Landreau, B.. A new proof of a theorem of van der Corput. Bull. London Math. Soc. 21 4 (1989), 366368.Google Scholar
[7] Munshi, R.. Inequalities for divisor functions. Ramanujan J. 25 2 (2011), 195201.Google Scholar
[8] Pokrovskiy, A.. A linear bound on the Manickam–Miklos–Singhi Conjecture. ArXiv e-prints (Aug. 2013).Google Scholar
[9] Soundararajan, K.. An inequality for multiplicative functions. J. Number Theory 41 2 (1992), 225230.Google Scholar
[10] Widder, D. V.. The Laplace Transform. Princeton Mathematical Series, v. 6 (Princeton University Press, Princeton, N. J., 1941).Google Scholar
[11] Wolke, D.. A new proof of a theorem of van der Corput. J. London Math. Soc. s2-5 4 (1972), 609612.Google Scholar